mixedmath

Explorations in math and programming
David Lowry-Duda



In this note, we produce a proof of Taylor's Theorem. As in many proofs of Taylor's Theorem, we begin with a curious start and then follow our noses forward.

Is this a new proof? I think so. But I wouldn't bet a lot of money on it. It's certainly new to me.

Is this a groundbreaking proof? No, not at all. But it's cute, and I like it.1 1Though I must admit that it is not my favorite proof of Taylor's Theorem.

We begin with the following simple observation. Suppose that $f$ is two times continuously differentiable. Then for any $t \neq 0$, we see that \begin{equation} f'(t) - f'(0) = \frac{f'(t) - f'(0)}{t} t. \end{equation} Integrating each side from $0$ to $x$, we find that \begin{equation} f(x) - f(0) - f'(0) x = \int_0^x \frac{f'(t) - f'(0)}{t} t dt. \end{equation} To interpret the integral on the right in a different way, we will use the mean value theorem for integrals.

Mean Value Theorem for Integrals Suppose that $g$ and $h$ are continuous functions, and that $h$ doesn't change sign in $[0, x]$. Then there is a $c \in [0, x]$ such that \begin{equation} \int_0^x g(t) h(t) dt = g(c) \int_0^x h(t) dt. \end{equation}
Suppose without loss of generality that $h(t)$ is nonnegative. Since $g$ is continuous on $[0, x]$, it attains its minimum $m$ and maximum $M$ on this interval. Thus \begin{equation} m \int_0^x h(t) dt \leq \int_0^x g(t)h(t)dt \leq M \int_0^x h(t) dt. \end{equation} Let $I = \int_0^x h(t) dt$. If $I = 0$ (or equivalently, if $h(t) \equiv 0$), then the theorem is trivially true, so suppose instead that $I \neq 0$. Then \begin{equation} m \leq \frac{1}{I} \int_0^x g(t) h(t) dt \leq M. \end{equation} By the intermediate value theorem, $g(t)$ attains every value between $m$ and $M$, and thus there exists some $c$ such that \begin{equation} g(c) = \frac{1}{I} \int_0^x g(t) h(t) dt. \end{equation} Rearranging proves the theorem.

For this application, let $g(t) = (f'(t) - f'(0))/t$ for $t \neq 0$, and $g(0) =f'{}'(0)$. The continuity of $g$ at $0$ is exactly the condition that $f'{}'(0)$exists. We also let $h(t) = t$.

For $x > 0$, it follows from the mean value theorem for integrals that there exists a $c \in [0, x]$ such that \begin{equation} \int_0^x \frac{f'(t) - f'(0)}{t} t dt = \frac{f'(c) - f'(0)}{c} \int_0^x t dt = \frac{f'(c) - f'(0)}{c} \frac{x^2}{2}. \end{equation} (Very similar reasoning applies for $x < 0$). Finally, by the mean value theorem (applied to $f'$), there exists a point $\xi \in (0, c)$ such that \begin{equation} f'{}'(\xi) = \frac{f'(c) - f'(0)}{c}. \end{equation} Putting this together, we have proved that there is a $\xi \in (0, x)$ such that \begin{equation} f(x) - f(0) - f'(0) x = f'{}'(\xi) \frac{x^2}{2}, \end{equation} which is one version of Taylor's Theorem with a linear approximating polynomial.

This approach generalizes. Suppose $f$ is a $(k+1)$ times continuously differentiable function, and begin with the trivial observation that \begin{equation} f^{(k)}(t) - f^{(k)}(0) = \frac{f^{(k)}(t) - f^{(k)}(0)}{t} t. \end{equation} Iteratively integrate $k$ times: first from $0$ to $t_1$, then from $0$ to $t_2$, and so on, with the $k$th interval being from $0$ to $t_k = x$.

Then the left hand side becomes \begin{equation} f(x) - \sum_{n = 0}^k f^{(n)}(0)\frac{x^n}{n!}, \end{equation} the difference between $f$ and its degree $k$ Taylor polynomial. The right hand side is \begin{equation}\label{eq:only}\underbrace{\int _0^{t_k = x} \cdots \int _0^{t _1}} _{k \text{ times}} \frac{f^{(k)}(t) - f^{(k)}(0)}{t} t \, dt \, dt _1 \cdots dt _{k-1}.\end{equation}

To handle this, we note the following variant of the mean value theorem for integrals.

Mean value theorem for iterated integrals Suppose that $g$ and $h$ are continuous functions, and that $h$ doesn't change sign in $[0, x]$. Then there is a $c \in [0, x]$ such that \begin{equation} \underbrace{\int_0^{t _k=x} \cdots \int _0^{t _1}} _{k \; \text{times}} g(t) h(t) dt =g(c) \underbrace{\int _0^{t _k=x} \cdots \int _0^{t _1}} _{k \; \text{times}} h(t) dt. \end{equation}

In fact, this can be proved in almost exactly the same way as in the single-integral version, so we do not repeat the proof.

With this theorem, there is a $c \in [0, x]$ such that we see that \eqref{eq:only} can be written as \begin{equation} \frac{f^{(k)}(c) - f^{(k)}(0)}{c} \underbrace{\int _0^{t _k = x} \cdots \int _0^{t _1}} _{k \; \text{times}} t \, dt \, dt _1 \cdots dt _{k-1}. \end{equation} By the mean value theorem, the factor in front of the integrals can be written as $f^{(k+1)}(\xi)$ for some $\xi \in (0, x)$. The integrals can be directly evaluated to be $x^{k+1}/(k+1)! $.

Thus overall, we find that \begin{equation} f(x) = \sum_{n = 0}^n f^{(n)}(0) \frac{x^n}{n!} + f^{(k+1)}(\xi) \frac{x^{k+1}}{(k+1)!} \end{equation} for some $\xi \in (0, x)$. Thus we have proved Taylor's Theorem (with Lagrange's error bound).


Leave a comment

Info on how to comment

To make a comment, please send an email using the button below. Your email address won't be shared (unless you include it in the body of your comment). If you don't want your real name to be used next to your comment, please specify the name you would like to use. If you want your name to link to a particular url, include that as well.

bold, italics, and plain text are allowed in comments. A reasonable subset of markdown is supported, including lists, links, and fenced code blocks. In addition, math can be formatted using $(inline math)$ or $$(your display equation)$$.

Please use plaintext email when commenting. See Plaintext Email and Comments on this site for more. Note also that comments are expected to be open, considerate, and respectful.

Comment via email